- PowerScore Staff
- Posts: 5972
- Joined: Mar 25, 2011
- Wed Mar 07, 2018 12:56 pm
#44233
Complete Question Explanation
(The complete setup for this game can be found here: lsat/viewtopic.php?t=9342)
The correct answer choice is (B)
This List question is easily answered by a systematic application of the rules.
Answer choice (A) is incorrect because N is selected but J is not selected.
Answer choice (B) is correct.
Answer choice (C) is incorrect because both M and J are selected.
Answer choice (D) is incorrect because both X and P are selected.
Answer choice (E) is incorrect because J and O are both selected yet O is played before J.
(The complete setup for this game can be found here: lsat/viewtopic.php?t=9342)
The correct answer choice is (B)
This List question is easily answered by a systematic application of the rules.
Answer choice (A) is incorrect because N is selected but J is not selected.
Answer choice (B) is correct.
Answer choice (C) is incorrect because both M and J are selected.
Answer choice (D) is incorrect because both X and P are selected.
Answer choice (E) is incorrect because J and O are both selected yet O is played before J.
Dave Killoran
PowerScore Test Preparation
Follow me on X/Twitter at http://twitter.com/DaveKilloran
My LSAT Articles: http://blog.powerscore.com/lsat/author/dave-killoran
PowerScore Podcast: http://www.powerscore.com/lsat/podcast/
PowerScore Test Preparation
Follow me on X/Twitter at http://twitter.com/DaveKilloran
My LSAT Articles: http://blog.powerscore.com/lsat/author/dave-killoran
PowerScore Podcast: http://www.powerscore.com/lsat/podcast/